All 20 basic level tasks. A group of tourists overcame a mountain pass. Biologists have discovered a variety of amoeba

Secondary general education

Line UMK G.K. Muravina. Algebra and the beginnings of mathematical analysis (10-11) (deep)

Line UMK Merzlyak. Algebra and the Beginnings of Analysis (10-11) (U)

Mathematics

Preparation for the exam in mathematics (profile level): tasks, solutions and explanations

We analyze tasks and solve examples with the teacher

The profile-level examination paper lasts 3 hours 55 minutes (235 minutes).

Minimum Threshold- 27 points.

The examination paper consists of two parts, which differ in content, complexity and number of tasks.

The defining feature of each part of the work is the form of tasks:

  • part 1 contains 8 tasks (tasks 1-8) with a short answer in the form of an integer or a final decimal fraction;
  • part 2 contains 4 tasks (tasks 9-12) with a short answer in the form of an integer or a final decimal fraction and 7 tasks (tasks 13-19) with a detailed answer (full record of the decision with the rationale for the actions performed).

Panova Svetlana Anatolievna, teacher of mathematics of the highest category of the school, work experience of 20 years:

“In order to get a school certificate, a graduate must pass two mandatory exams in the form of the Unified State Examination, one of which is mathematics. In accordance with the Concept for the Development of Mathematical Education in the Russian Federation, the Unified State Examination in mathematics is divided into two levels: basic and specialized. Today we will consider options for the profile level.

Task number 1- checks the ability of USE participants to apply the skills acquired in the course of 5-9 grades in elementary mathematics in practical activities. The participant must have computational skills, be able to work with rational numbers, be able to round decimal fractions, be able to convert one unit of measurement to another.

Example 1 In the apartment where Petr lives, a cold water meter (meter) was installed. On the first of May, the meter showed an consumption of 172 cubic meters. m of water, and on the first of June - 177 cubic meters. m. What amount should Peter pay for cold water for May, if the price of 1 cu. m of cold water is 34 rubles 17 kopecks? Give your answer in rubles.

Decision:

1) Find the amount of water spent per month:

177 - 172 = 5 (cu m)

2) Find how much money will be paid for the spent water:

34.17 5 = 170.85 (rub)

Answer: 170,85.


Task number 2- is one of the simplest tasks of the exam. The majority of graduates successfully cope with it, which indicates the possession of the definition of the concept of function. Task type No. 2 according to the requirements codifier is a task for using the acquired knowledge and skills in practical activities and everyday life. Task No. 2 consists of describing, using functions, various real relationships between quantities and interpreting their graphs. Task number 2 tests the ability to extract information presented in tables, diagrams, graphs. Graduates need to be able to determine the value of a function by the value of the argument with various ways of specifying the function and describe the behavior and properties of the function according to its graph. It is also necessary to be able to find the largest or smallest value from the function graph and build graphs of the studied functions. The mistakes made are of a random nature in reading the conditions of the problem, reading the diagram.

#ADVERTISING_INSERT#

Example 2 The figure shows the change in the exchange value of one share of a mining company in the first half of April 2017. On April 7, the businessman purchased 1,000 shares of this company. On April 10, he sold three-quarters of the purchased shares, and on April 13 he sold all the remaining ones. How much did the businessman lose as a result of these operations?


Decision:

2) 1000 3/4 = 750 (shares) - make up 3/4 of all purchased shares.

6) 247500 + 77500 = 325000 (rubles) - the businessman received after the sale of 1000 shares.

7) 340,000 - 325,000 = 15,000 (rubles) - the businessman lost as a result of all operations.

Answer: 15000.

Task number 3- is a task of the basic level of the first part, it checks the ability to perform actions with geometric shapes according to the content of the course "Planimetry". Task 3 tests the ability to calculate the area of ​​a figure on checkered paper, the ability to calculate degree measures of angles, calculate perimeters, etc.

Example 3 Find the area of ​​a rectangle drawn on checkered paper with a cell size of 1 cm by 1 cm (see figure). Give your answer in square centimeters.

Decision: To calculate the area of ​​this figure, you can use the Peak formula:

To calculate the area of ​​this rectangle, we use the Peak formula:

S= B +

G
2
where V = 10, G = 6, therefore

S = 18 +

6
2
Answer: 20.

See also: Unified State Examination in Physics: solving vibration problems

Task number 4- the task of the course "Probability Theory and Statistics". The ability to calculate the probability of an event in the simplest situation is tested.

Example 4 There are 5 red and 1 blue dots on the circle. Determine which polygons are larger: those with all red vertices, or those with one of the blue vertices. In your answer, indicate how many more of one than the other.

Decision: 1) We use the formula for the number of combinations from n elements by k:

all of whose vertices are red.

3) One pentagon with all red vertices.

4) 10 + 5 + 1 = 16 polygons with all red vertices.

whose vertices are red or with one blue vertex.

whose vertices are red or with one blue vertex.

8) One hexagon whose vertices are red with one blue vertex.

9) 20 + 15 + 6 + 1 = 42 polygons that have all red vertices or one blue vertex.

10) 42 - 16 = 26 polygons that use the blue dot.

11) 26 - 16 = 10 polygons - how many polygons, in which one of the vertices is a blue dot, are more than polygons, in which all vertices are only red.

Answer: 10.

Task number 5- the basic level of the first part tests the ability to solve the simplest equations (irrational, exponential, trigonometric, logarithmic).

Example 5 Solve Equation 2 3 + x= 0.4 5 3 + x .

Decision. Divide both sides of this equation by 5 3 + X≠ 0, we get

2 3 + x = 0.4 or 2 3 + X = 2 ,
5 3 + X 5 5

whence it follows that 3 + x = 1, x = –2.

Answer: –2.

Task number 6 in planimetry for finding geometric quantities (lengths, angles, areas), modeling real situations in the language of geometry. The study of the constructed models using geometric concepts and theorems. The source of difficulties is, as a rule, ignorance or incorrect application of the necessary theorems of planimetry.

Area of ​​a triangle ABC equals 129. DE- median line parallel to side AB. Find the area of ​​the trapezoid ABED.


Decision. Triangle CDE similar to a triangle CAB at two corners, since the corner at the vertex C general, angle CDE equal to the angle CAB as the corresponding angles at DE || AB secant AC. As DE is the middle line of the triangle by the condition, then by the property of the middle line | DE = (1/2)AB. So the similarity coefficient is 0.5. The areas of similar figures are related as the square of the similarity coefficient, so

Hence, S ABED = S Δ ABCS Δ CDE = 129 – 32,25 = 96,75.

Task number 7- checks the application of the derivative to the study of the function. For successful implementation, a meaningful, non-formal possession of the concept of a derivative is necessary.

Example 7 To the graph of the function y = f(x) at the point with the abscissa x 0 a tangent is drawn, which is perpendicular to the straight line passing through the points (4; 3) and (3; -1) of this graph. Find f′( x 0).

Decision. 1) Let's use the equation of a straight line passing through two given points and find the equation of a straight line passing through points (4; 3) and (3; -1).

(yy 1)(x 2 – x 1) = (xx 1)(y 2 – y 1)

(y – 3)(3 – 4) = (x – 4)(–1 – 3)

(y – 3)(–1) = (x – 4)(–4)

y + 3 = –4x+ 16| · (-one)

y – 3 = 4x – 16

y = 4x– 13, where k 1 = 4.

2) Find the slope of the tangent k 2 which is perpendicular to the line y = 4x– 13, where k 1 = 4, according to the formula:

3) The slope of the tangent is the derivative of the function at the point of contact. Means, f′( x 0) = k 2 = –0,25.

Answer: –0,25.

Task number 8- checks the knowledge of elementary stereometry among the participants of the exam, the ability to apply formulas for finding surface areas and volumes of figures, dihedral angles, compare the volumes of similar figures, be able to perform actions with geometric figures, coordinates and vectors, etc.

The volume of a cube circumscribed around a sphere is 216. Find the radius of the sphere.


Decision. 1) V cube = a 3 (where a is the length of the edge of the cube), so

a 3 = 216

a = 3 √216

2) Since the sphere is inscribed in a cube, it means that the length of the diameter of the sphere is equal to the length of the edge of the cube, therefore d = a, d = 6, d = 2R, R = 6: 2 = 3.

Task number 9- requires the graduate to transform and simplify algebraic expressions. Task No. 9 of an increased level of complexity with a short answer. Tasks from the section "Calculations and transformations" in the USE are divided into several types:

    transformations of numerical rational expressions;

    transformations of algebraic expressions and fractions;

    transformations of numerical/letter irrational expressions;

    actions with degrees;

    transformation of logarithmic expressions;

  1. conversion of numeric/letter trigonometric expressions.

Example 9 Calculate tgα if it is known that cos2α = 0.6 and

< α < π.
4

Decision. 1) Let's use the double argument formula: cos2α = 2 cos 2 α - 1 and find

tan 2 α = 1 – 1 = 1 – 1 = 10 – 1 = 5 – 1 = 1 1 – 1 = 1 = 0,25.
cos 2 α 0,8 8 4 4 4

Hence, tan 2 α = ± 0.5.

3) By condition

< α < π,
4

hence α is the angle of the second quarter and tgα< 0, поэтому tgα = –0,5.

Answer: –0,5.

#ADVERTISING_INSERT# Task number 10- checks the ability of students to use the acquired early knowledge and skills in practical activities and everyday life. We can say that these are problems in physics, and not in mathematics, but all the necessary formulas and quantities are given in the condition. The tasks are reduced to solving a linear or quadratic equation, or a linear or quadratic inequality. Therefore, it is necessary to be able to solve such equations and inequalities, and determine the answer. The answer must be in the form of a whole number or a final decimal fraction.

Two bodies of mass m= 2 kg each, moving at the same speed v= 10 m/s at an angle of 2α to each other. The energy (in joules) released during their absolutely inelastic collision is determined by the expression Q = mv 2 sin 2 α. At what smallest angle 2α (in degrees) must the bodies move so that at least 50 joules are released as a result of the collision?
Decision. To solve the problem, we need to solve the inequality Q ≥ 50, on the interval 2α ∈ (0°; 180°).

mv 2 sin 2 α ≥ 50

2 10 2 sin 2 α ≥ 50

200 sin2α ≥ 50

Since α ∈ (0°; 90°), we will only solve

We represent the solution of the inequality graphically:


Since by assumption α ∈ (0°; 90°), it means that 30° ≤ α< 90°. Получили, что наименьший угол α равен 30°, тогда наименьший угол 2α = 60°.

Task number 11- is typical, but it turns out to be difficult for students. The main source of difficulties is the construction of a mathematical model (drawing up an equation). Task number 11 tests the ability to solve word problems.

Example 11. During spring break, 11-grader Vasya had to solve 560 training problems to prepare for the exam. On March 18, on the last day of school, Vasya solved 5 problems. Then every day he solved the same number of problems more than the previous day. Determine how many problems Vasya solved on April 2 on the last day of vacation.

Decision: Denote a 1 = 5 - the number of tasks that Vasya solved on March 18, d– daily number of tasks solved by Vasya, n= 16 - the number of days from March 18 to April 2 inclusive, S 16 = 560 - the total number of tasks, a 16 - the number of tasks that Vasya solved on April 2. Knowing that every day Vasya solved the same number of tasks more than the previous day, then you can use the formulas for finding the sum of an arithmetic progression:

560 = (5 + a 16) 8,

5 + a 16 = 560: 8,

5 + a 16 = 70,

a 16 = 70 – 5

a 16 = 65.

Answer: 65.

Task number 12- check students' ability to perform actions with functions, be able to apply the derivative to the study of the function.

Find the maximum point of a function y= 10ln( x + 9) – 10x + 1.

Decision: 1) Find the domain of the function: x + 9 > 0, x> –9, that is, x ∈ (–9; ∞).

2) Find the derivative of the function:

4) The found point belongs to the interval (–9; ∞). We define the signs of the derivative of the function and depict the behavior of the function in the figure:


The desired maximum point x = –8.

Download for free the work program in mathematics to the line of UMK G.K. Muravina, K.S. Muravina, O.V. Muravina 10-11 Download free algebra manuals

Task number 13- an increased level of complexity with a detailed answer, which tests the ability to solve equations, the most successfully solved among tasks with a detailed answer of an increased level of complexity.

a) Solve the equation 2log 3 2 (2cos x) – 5log 3 (2cos x) + 2 = 0

b) Find all the roots of this equation that belong to the segment.

Decision: a) Let log 3 (2cos x) = t, then 2 t 2 – 5t + 2 = 0,


log3(2cos x) = 2
2cos x = 9
cos x = 4,5 ⇔ because |cos x| ≤ 1,
log3(2cos x) = 1 2cos x = √3 cos x = √3
2 2
then cos x = √3
2

x = π + 2π k
6
x = – π + 2π k, kZ
6

b) Find the roots lying on the segment .


It can be seen from the figure that the given segment has roots

11π and 13π .
6 6
Answer: a) π + 2π k; – π + 2π k, kZ; b) 11π ; 13π .
6 6 6 6
Task number 14- advanced level refers to the tasks of the second part with a detailed answer. The task tests the ability to perform actions with geometric shapes. The task contains two items. In the first paragraph, the task must be proved, and in the second paragraph, it must be calculated.

The diameter of the circle of the base of the cylinder is 20, the generatrix of the cylinder is 28. The plane intersects its bases along chords of length 12 and 16. The distance between the chords is 2√197.

a) Prove that the centers of the bases of the cylinder lie on the same side of this plane.

b) Find the angle between this plane and the plane of the base of the cylinder.

Decision: a) A chord of length 12 is at a distance = 8 from the center of the base circle, and a chord of length 16, similarly, is at a distance of 6. Therefore, the distance between their projections on a plane parallel to the bases of the cylinders is either 8 + 6 = 14, or 8 − 6 = 2.

Then the distance between chords is either

= = √980 = = 2√245

= = √788 = = 2√197.

According to the condition, the second case was realized, in which the projections of the chords lie on one side of the axis of the cylinder. This means that the axis does not intersect this plane within the cylinder, that is, the bases lie on one side of it. What needed to be proven.

b) Let's denote the centers of the bases as O 1 and O 2. Let us draw from the center of the base with a chord of length 12 the perpendicular bisector to this chord (it has a length of 8, as already noted) and from the center of the other base to another chord. They lie in the same plane β perpendicular to these chords. Let's call the midpoint of the smaller chord B, greater than A, and the projection of A onto the second base H (H ∈ β). Then AB,AH ∈ β and, therefore, AB,AH are perpendicular to the chord, that is, the line of intersection of the base with the given plane.

So the required angle is

∠ABH = arctan AH = arctg 28 = arctg14.
BH 8 – 6

Task number 15- an increased level of complexity with a detailed answer, checks the ability to solve inequalities, the most successfully solved among tasks with a detailed answer of an increased level of complexity.

Example 15 Solve the inequality | x 2 – 3x| log 2 ( x + 1) ≤ 3xx 2 .

Decision: The domain of definition of this inequality is the interval (–1; +∞). Consider three cases separately:

1) Let x 2 – 3x= 0, i.e. X= 0 or X= 3. In this case, this inequality becomes true, therefore, these values ​​are included in the solution.

2) Let now x 2 – 3x> 0, i.e. x∈ (–1; 0) ∪ (3; +∞). In this case, this inequality can be rewritten in the form ( x 2 – 3x) log 2 ( x + 1) ≤ 3xx 2 and divide by a positive expression x 2 – 3x. We get log 2 ( x + 1) ≤ –1, x + 1 ≤ 2 –1 , x≤ 0.5 -1 or x≤ -0.5. Taking into account the domain of definition, we have x ∈ (–1; –0,5].

3) Finally, consider x 2 – 3x < 0, при этом x∈ (0; 3). In this case, the original inequality will be rewritten in the form (3 xx 2) log 2 ( x + 1) ≤ 3xx 2. After dividing by a positive expression 3 xx 2 , we get log 2 ( x + 1) ≤ 1, x + 1 ≤ 2, x≤ 1. Taking into account the area, we have x ∈ (0; 1].

Combining the obtained solutions, we obtain x ∈ (–1; –0.5] ∪ ∪ {3}.

Answer: (–1; –0.5] ∪ ∪ {3}.

Task number 16- advanced level refers to the tasks of the second part with a detailed answer. The task tests the ability to perform actions with geometric shapes, coordinates and vectors. The task contains two items. In the first paragraph, the task must be proved, and in the second paragraph, it must be calculated.

In an isosceles triangle ABC with an angle of 120° at the vertex A, a bisector BD is drawn. Rectangle DEFH is inscribed in triangle ABC so that side FH lies on segment BC and vertex E lies on segment AB. a) Prove that FH = 2DH. b) Find the area of ​​the rectangle DEFH if AB = 4.

Decision: a)


1) ΔBEF - rectangular, EF⊥BC, ∠B = (180° - 120°) : 2 = 30°, then EF = BE due to the property of the leg opposite the angle of 30°.

2) Let EF = DH = x, then BE = 2 x, BF = x√3 by the Pythagorean theorem.

3) Since ΔABC is isosceles, then ∠B = ∠C = 30˚.

BD is the bisector of ∠B, so ∠ABD = ∠DBC = 15˚.

4) Consider ΔDBH - rectangular, because DH⊥BC.

2x = 4 – 2x
2x(√3 + 1) 4
1 = 2 – x
√3 + 1 2

√3 – 1 = 2 – x

x = 3 – √3

EF = 3 - √3

2) S DEFH = ED EF = (3 - √3 ) 2(3 - √3 )

S DEFH = 24 - 12√3.

Answer: 24 – 12√3.


Task number 17- a task with a detailed answer, this task tests the application of knowledge and skills in practical activities and everyday life, the ability to build and explore mathematical models. This task is a text task with economic content.

Example 17. The deposit in the amount of 20 million rubles is planned to be opened for four years. At the end of each year, the bank increases the deposit by 10% compared to its size at the beginning of the year. In addition, at the beginning of the third and fourth years, the depositor annually replenishes the deposit by X million rubles, where X - whole number. Find the highest value X, at which the bank will add less than 17 million rubles to the deposit in four years.

Decision: At the end of the first year, the contribution will be 20 + 20 · 0.1 = 22 million rubles, and at the end of the second - 22 + 22 · 0.1 = 24.2 million rubles. At the beginning of the third year, the contribution (in million rubles) will be (24.2 + X), and at the end - (24.2 + X) + (24,2 + X) 0.1 = (26.62 + 1.1 X). At the beginning of the fourth year, the contribution will be (26.62 + 2.1 X), and at the end - (26.62 + 2.1 X) + (26,62 + 2,1X) 0.1 = (29.282 + 2.31 X). By condition, you need to find the largest integer x for which the inequality

(29,282 + 2,31x) – 20 – 2x < 17

29,282 + 2,31x – 20 – 2x < 17

0,31x < 17 + 20 – 29,282

0,31x < 7,718

x < 7718
310
x < 3859
155
x < 24 139
155

The largest integer solution to this inequality is the number 24.

Answer: 24.


Task number 18- a task of an increased level of complexity with a detailed answer. This task is intended for competitive selection to universities with increased requirements for the mathematical preparation of applicants. A task of a high level of complexity is not a task for applying one solution method, but for a combination of different methods. For the successful completion of task 18, in addition to solid mathematical knowledge, a high level of mathematical culture is also required.

At what a system of inequalities

x 2 + y 2 ≤ 2aya 2 + 1
y + a ≤ |x| – a

has exactly two solutions?

Decision: This system can be rewritten as

x 2 + (ya) 2 ≤ 1
y ≤ |x| – a

If we draw on the plane the set of solutions to the first inequality, we get the interior of a circle (with boundary) of radius 1 centered at the point (0, a). The set of solutions of the second inequality is the part of the plane that lies under the graph of the function y = | x| – a, and the latter is the graph of the function
y = | x| , shifted down by a. The solution of this system is the intersection of the solution sets of each of the inequalities.

Consequently, this system will have two solutions only in the case shown in Fig. one.


The points of contact between the circle and the lines will be the two solutions of the system. Each of the straight lines is inclined to the axes at an angle of 45°. So the triangle PQR- rectangular isosceles. Dot Q has coordinates (0, a), and the point R– coordinates (0, – a). In addition, cuts PR and PQ are equal to the circle radius equal to 1. Hence,

QR= 2a = √2, a = √2 .
2
Answer: a = √2 .
2


Task number 19- a task of an increased level of complexity with a detailed answer. This task is intended for competitive selection to universities with increased requirements for the mathematical preparation of applicants. A task of a high level of complexity is not a task for applying one solution method, but for a combination of different methods. For the successful completion of task 19, it is necessary to be able to search for a solution, choosing various approaches from among the known ones, modifying the studied methods.

Let be sn sum P members of an arithmetic progression ( a p). It is known that S n + 1 = 2n 2 – 21n – 23.

a) Give the formula P th member of this progression.

b) Find the smallest modulo sum S n.

c) Find the smallest P, at which S n will be the square of an integer.

Decision: a) Obviously, a n = S nS n- one . Using this formula, we get:

S n = S (n – 1) + 1 = 2(n – 1) 2 – 21(n – 1) – 23 = 2n 2 – 25n,

S n – 1 = S (n – 2) + 1 = 2(n – 1) 2 – 21(n – 2) – 23 = 2n 2 – 25n+ 27

means, a n = 2n 2 – 25n – (2n 2 – 29n + 27) = 4n – 27.

B) because S n = 2n 2 – 25n, then consider the function S(x) = | 2x 2 – 25x|. Her graph can be seen in the figure.


It is obvious that the smallest value is reached at the integer points located closest to the zeros of the function. Obviously these are points. X= 1, X= 12 and X= 13. Since, S(1) = |S 1 | = |2 – 25| = 23, S(12) = |S 12 | = |2 144 – 25 12| = 12, S(13) = |S 13 | = |2 169 – 25 13| = 13, then the smallest value is 12.

c) It follows from the previous paragraph that sn positive since n= 13. Since S n = 2n 2 – 25n = n(2n– 25), then the obvious case when this expression is a perfect square is realized when n = 2n- 25, that is, with P= 25.

It remains to check the values ​​​​from 13 to 25:

S 13 = 13 1, S 14 = 14 3, S 15 = 15 5, S 16 = 16 7, S 17 = 17 9, S 18 = 18 11, S 19 = 19 13 S 20 = 20 13, S 21 = 21 17, S 22 = 22 19, S 23 = 23 21, S 24 = 24 23.

It turns out that for smaller values P full square is not achieved.

Answer: a) a n = 4n- 27; b) 12; c) 25.

________________

*Since May 2017, the DROFA-VENTANA joint publishing group has been part of the Russian Textbook Corporation. The corporation also included the Astrel publishing house and the LECTA digital educational platform. Alexander Brychkin, a graduate of the Financial Academy under the Government of the Russian Federation, candidate of economic sciences, head of innovative projects of the DROFA publishing house in the field of digital education (electronic forms of textbooks, Russian Electronic School, LECTA digital educational platform) has been appointed General Director. Prior to joining the DROFA publishing house, he held the position of Vice President for Strategic Development and Investments of the EKSMO-AST publishing holding. Today, the Russian Textbook Publishing Corporation has the largest portfolio of textbooks included in the Federal List - 485 titles (approximately 40%, excluding textbooks for correctional schools). The corporation's publishing houses own the sets of textbooks in physics, drawing, biology, chemistry, technology, geography, astronomy, most in demand by Russian schools - areas of knowledge that are needed to develop the country's production potential. The corporation's portfolio includes textbooks and teaching aids for elementary schools awarded the President's Prize in Education. These are textbooks and manuals on subject areas that are necessary for the development of the scientific, technical and industrial potential of Russia.

Task 20 Basic level of the exam

1) A snail crawls 4 m up a tree in a day, and slides 1 m in a night. The height of a tree is 13 m. In how many days will a snail crawl to the top of a tree for the first time? (4-1 \u003d 3, the morning of the 4th day will be at a height of 9m, and 4m will crawl in a day.Answer: 4 )

2) A snail crawls 4 m up a tree in a day, and slides 3 m in a night. The height of a tree is 10 m. In how many days will a snail crawl to the top of a tree for the first time? Answer: 7

3) A snail climbs 3 m up a tree in a day, and descends 2 m in a night. The height of a tree is 10 m. How many days will a snail climb to the top of a tree? Answer: 8

4) Cross lines of red, yellow and green are marked on the stick. If you cut a stick along the red lines, you get 15 pieces, if along the yellow lines - 5 pieces, and if along the green lines - 7 pieces. How many pieces do you get if you cut a stick along the lines of all three colors ? (If you cut a stick along red lines, you get 15 pieces, therefore, lines - 14. If you saw a stick along yellow lines - 5 pieces, therefore, lines - 4. If you saw it along green lines - 7 pieces, lines - 6. Total lines: 14 + 4 + 6 = 24 lines. Answer:25 )

5) On the stick are marked transverse lines of red, yellow and green. If you saw the stick along the red lines, you get 5 pieces, if along the yellow lines - 7 pieces, and if along the green lines - 11 pieces. How many pieces will you get if you cut a stick along the lines of all three colors? Answer : 21

6) Transverse lines of red, yellow and green are marked on the stick. If you cut a stick along the red lines, you get 10 pieces, if along the yellow lines - 8 pieces, if along the green lines - 8 pieces. How many pieces will you get if you cut a stick along the lines of all three colors? Answer : 24

7) In the exchange office, you can perform one of two operations:

For 2 gold coins, get 3 silver and one copper;

For 5 silver coins, get 3 gold and one copper.

Nicholas had only silver coins. After several visits to the exchange office, he had fewer silver coins, no gold coins, but 50 copper coins appeared. By how much did Nicholas's number of silver coins decrease? Answer: 10

8) At the exchange office, you can perform one of two operations:

· for 2 gold coins get 3 silver and one copper;

· For 5 silver coins, get 3 gold and one copper.

Nicholas had only silver coins. After several visits to the exchange office, he had fewer silver coins, no gold coins, but 100 copper coins appeared. By how much did Nicholas's number of silver coins decrease?? Answer: 20

9) In the exchange office, you can perform one of two operations:

1) for 3 gold coins get 4 silver and one copper;

2) for 6 silver coins, get 4 gold and one copper.

Nikola had only silver coins. After visiting the exchange office, he had fewer silver coins, no gold coins, but 35 copper coins appeared. By how much did Nikola's number of silver coins decrease? Answer: 10

10) In the exchange office, you can perform one of two operations:

1) for 3 gold coins get 4 silver and one copper;

2) for 7 silver coins, get 4 gold and one copper.

Nikola had only silver coins. After visiting the exchange office, he had fewer silver coins, no gold coins, but 42 copper coins appeared. By how much did Nikola's number of silver coins decrease? Answer: 30

11) In the exchange office, you can perform one of two operations:

1) for 4 gold coins get 5 silver and one copper;

2) for 8 silver coins, get 5 gold and one copper.

Nicholas had only silver coins. After several visits to the exchange office, he had fewer silver coins, no gold coins, but 45 copper coins appeared. By how much did Nicholas's number of silver coins decrease? Answer: 35

12) There are 50 mushrooms in the basket: mushrooms and milk mushrooms. It is known that among any 28 mushrooms there is at least one camelina, and among any 24 mushrooms at least one mushroom. How many mushrooms are in the basket? ( (50-28)+1=23 - must be redheads. (50-24)+1=27 - must be gruzdey. Answer: mushrooms in the basket 27 .)

13) There are 40 mushrooms in the basket: mushrooms and milk mushrooms. It is known that among any 17 mushrooms there is at least one camelina, and among any 25 mushrooms at least one mushroom. How many mushrooms are in the basket? ( According to the condition of the problem: (40-17)+1=24 - must be redheads. (40-25)+1=16 24 .)

14) the basket contains 30 mushrooms: mushrooms and milk mushrooms. It is known that among any 12 mushrooms there is at least one camelina, and among any 20 mushrooms at least one mushroom. How many mushrooms are in the basket? (According to the condition of the problem: (30-12)+1=19 - must be redheads. (30-20)+1=11 - must be gruzdey. Answer: saffron milk caps in a basket 19 .)

15) There are 45 mushrooms in the basket: mushrooms and milk mushrooms. It is known that among any 23 mushrooms there is at least one camelina, and among any 24 mushrooms at least one mushroom. How many mushrooms are in the basket? ( According to the condition of the problem: (45-23)+1=23 - must be redheads. (45-24)+1=22 - must be gruzdey. Answer: saffron milk caps in a basket 23 .)

16) There are 25 mushrooms in the basket: mushrooms and milk mushrooms. It is known that among any 11 mushrooms there is at least one camelina, and among any 16 mushrooms at least one mushroom. How many mushrooms are in the basket? ( Since among any 11 mushrooms at least one is a mushroom, then there are no more than 10 mushrooms. Since among any 16 mushrooms at least one is a mushroom, then there are no more than 15 mushrooms. And since there are 25 mushrooms in the basket, there are exactly 10 mushrooms, and Ryzhikov exactlyAnswer:15.

17) The owner agreed with the workers that they would dig a well for him on the following conditions: for the first meter he would pay them 4200 rubles, and for each next meter - 1300 rubles more than for the previous one. How much money will the owner have to pay the workers if they dig a well 11 meters deep ?(Answer: 117700)

18) The owner agreed with the workers that they would dig a well for him on the following conditions: for the first meter he would pay them 3,700 rubles, and for each next meter - 1,700 rubles more than for the previous one. How much money will the owner have to pay the workers if they dig a well 8 meters deep? ( 77200 )

19) The owner agreed with the workers that they are digging a well on the following terms: for the first meter he will pay them 3,500 rubles, and for each next meter - 1,600 rubles more than for the previous one. How much money will the owner have to pay the workers if they dig a well 9 meters deep? ( 89100 )

20) The owner agreed with the workers that they would dig a well for him on the following conditions: for the first meter he would pay them 3,900 rubles, and for each next meter he would pay 1,200 rubles more than for the previous one. How many rubles will the owner have to pay to the workers if they dig a well 6 meters deep? (41400)

21) The trainer advised Andrey to spend 15 minutes on the treadmill on the first day of classes, and on each next lesson to increase the time spent on the treadmill by 7 minutes. How many sessions will Andrey spend on the treadmill for a total of 2 hours and 25 minutes if he follows the advice of the trainer? ( 5 )

22) The coach advised Andrey to spend 22 minutes on the treadmill on the first day of training, and on each next session to increase the time spent on the treadmill by 4 minutes until it reaches 60 minutes, and then continue to train for 60 minutes every day. In how many sessions, starting from the first one, Andrey will spend 4 hours and 48 minutes on the treadmill? ( 8 )

23) There are 24 seats in the first row of the cinema hall, and in each next row there are 2 more than in the previous one. How many seats are in the eighth row? ( 38 )

24) The doctor prescribed the patient to take the medicine according to the following scheme: on the first day he should take 3 drops, and on each next day - 3 drops more than on the previous one. Having taken 30 drops, he drinks 30 drops of the medicine for another 3 days, and then reduces the intake by 3 drops daily. How many vials of medicine should a patient buy for the entire course of treatment if each contains 20 ml of medicine (which is 250 drops)? (2) the sum of an arithmetic progression with the first term equal to 3, the difference equal to 3 and the last term equal to 30.; 165 + 90 + 135 = 390 drops; 3+ 3(n-1)=30; n=10 and 27- 3(n-1)=3; n=9

25) The doctor prescribed the patient to take the medicine according to the following scheme: on the first day he should take 20 drops, and on each next day - 3 drops more than on the previous one. After 15 days of taking the patient takes a break of 3 days and continues to take the medicine according to the reverse scheme: on the 19th day he takes the same number of drops as on the 15th day, and then reduces the dose by 3 drops daily until the dosage becomes less than 3 drops per day. How many vials of medicine should a patient buy for the entire course of treatment if each contains 200 drops? ( 7 ) drinks 615 + 615 + 55 = 1285; 1285: 200 = 6.4

26) In a household appliance store, sales of refrigerators are seasonal. In January, 10 refrigerators were sold, and in the next three months, 10 refrigerators were sold. Since May, sales have increased by 15 units compared to the previous month. Since September, sales began to decrease by 15 refrigerators every month compared to the previous month. How many refrigerators did the store sell in a year? (360) (5*10+2*25+2*40+2*55+70=360

27) On the surface of the globe, 12 parallels and 22 meridians were drawn with a felt-tip pen. Into how many parts did the drawn lines divide the surface of the globe?

A meridian is an arc of a circle connecting the North and South Poles. A parallel is a circle lying in a plane parallel to the plane of the equator. (13 22=286)

28) On the surface of the globe, 17 parallels and 24 meridians were drawn with a felt-tip pen. Into how many parts did the drawn lines divide the surface of the globe? A meridian is an arc of a circle connecting the North and South Poles. A parallel is a circle lying in a plane parallel to the plane of the equator. (18 24 =432)

29) What is the smallest number of consecutive numbers you need to take so that their product is divisible by 7? (2) If the condition of the problem sounded like this: “What is the smallest number of consecutive numbers you need to take so that their product guaranteed divisible by 7? Then it would be necessary to take seven consecutive numbers.

30) What is the smallest number of consecutive numbers you need to take so that their product is divisible by 9? (2)

31) The product of ten consecutive numbers is divided by 7. What can be the remainder? (0) Among 10 consecutive numbers, one of them will necessarily be divisible by 7, so the product of these numbers is a multiple of seven. Therefore, the remainder when divided by 7 is zero.

32) The grasshopper jumps along the coordinate line in any direction for a unit segment per jump. How many different points on the coordinate line are there that the grasshopper can reach after making exactly 6 jumps, starting from the origin? ( the grasshopper can end up at points: -6, -4, -2, 0, 2, 4 and 6; only 7 points.)

33) The grasshopper jumps along the coordinate line in any direction for a unit segment per jump. How many different points on the coordinate line are there that the grasshopper can reach after making exactly 12 jumps, starting from the origin? ( the grasshopper can end up at points: -12, -10, -8, -6, -4, -2, 0, 2, 4, 6, 8, 10 and 12; total 13 points.)

34) The grasshopper jumps along the coordinate line in any direction for a unit segment per jump. How many different points on the coordinate line are there that the grasshopper can reach after making exactly 11 jumps, starting from the origin? (may appear at points: -11, -9, -7, -5, -3, -1, 1, 3, 5, 7, 9 and 11; 12 points in total.)

35) The grasshopper jumps along the coordinate line in any direction for a unit segment per jump. How many different points on the coordinate line are there that the grasshopper can reach after making exactly 8 jumps, starting from the origin?

Note that the grasshopper can only end up at points with even coordinates, since the number of jumps it makes is even. The maximum grasshopper can be at points, the module of which does not exceed eight. Thus, the grasshopper can end up at the points: -8, -6,-2 ; −4, 0.2 , 4, 6, 8 total 9 points.

Problem #5922.

The owner agreed with the workers that they were digging a well on the following terms: for the first meter he would pay them 3,500 rubles, and for each next meter - 1,600 rubles more than for the previous one. How much money will the owner have to pay the workers if they dig a well 9 meters deep?

Since the payment for each next meter differs from the payment for the previous one by the same number, we have before us.

In this progression - the payment for the first meter, - the difference in payment for each subsequent meter, - the number of working days.

The sum of the members of an arithmetic progression is found by the formula:

Substitute the data of the problem in this formula.

Answer: 89100.

Problem #5943.

In the exchange office, you can perform one of two operations:

· for 2 gold coins get 3 silver and one copper;

· For 5 silver coins, get 3 gold and one copper.

Nicholas had only silver coins. After several visits to the exchange office, he had fewer silver coins, no gold coins, but 100 copper coins appeared. By how much did Nicholas's number of silver coins decrease??

Problem #5960.

The grasshopper jumps along the coordinate line in any direction for a unit segment per jump. How many different points on the coordinate line are there that the grasshopper can reach after making exactly 5 jumps, starting from the origin?

If the grasshopper makes five jumps in one direction (right or left), then it will end up at points with coordinates 5 or -5:

Note that the grasshopper can jump both to the right and to the left. If he makes 1 jump to the right and 4 jumps to the left (for a total of 5 jumps), he will end up at the point with coordinate -3. Similarly, if the grasshopper makes 1 jump to the left and 4 jumps to the right (for a total of 5 jumps), then it will end up at the point with coordinate 3:

If the grasshopper makes 2 jumps to the right and 3 jumps to the left (for a total of 5 jumps), it will end up at the point with coordinate -1. Similarly, if the grasshopper makes 2 jumps to the left and 3 jumps to the right (for a total of 5 jumps), then it will end up at the point with coordinate 1:


Note that if the total number of jumps is odd, then the grasshopper will not return to the origin, that is, it can only hit points with odd coordinates:


There are only 6 of these points.

If the number of jumps were even, then the grasshopper could return to the origin and all points on the coordinate line that it could hit would have even coordinates.

Answer: 6

Problem #5990

A snail climbs up a tree 2 m in a day, and slides down 1 m in a night. The height of the tree is 9 m. How many days will it take for the snail to crawl to the top of the tree?

Note that in this problem one should distinguish between the concept of "day" and the concept of "day".

The question asks exactly how much days the snail will crawl to the top of the tree.

In one day the snail climbs 2 m, and in one day the snail rises to 1 m (it rises by 2 m during the day, and then descends by 1 m during the night).

For 7 days the snail rises to 7 meters. That is, on the morning of the 8th day, she will have to crawl to the top of 2 m. And on the eighth day she will overcome this distance.

Answer: 8 days.

Task number 6010.

All entrances of the house have the same number of floors, and each floor has the same number of apartments. At the same time, the number of floors in the house is greater than the number of apartments per floor, the number of apartments per floor is greater than the number of entrances, and the number of entrances is more than one. How many floors are there in a building if there are 105 apartments in total?

To find the number of apartments in a house, you need to multiply the number of apartments per floor ( ) by the number of floors ( ) and multiply by the number of entrances ( ).

That is, we need to find ( ) based on the following conditions:

(1)

The last inequality reflects the condition "the number of floors in the building is greater than the number of apartments per floor, the number of apartments per floor is greater than the number of entrances, and the number of entrances is more than one."

That is, ( ) is the largest number.

Let's factorize 105 into prime factors:

Taking into account condition (1), .

Answer: 7.

Problem #6036.

There are 30 mushrooms in the basket: mushrooms and milk mushrooms. It is known that among any 12 mushrooms there is at least one camelina, and among any 20 mushrooms at least one mushroom. How many mushrooms are in the basket?

As among any 12 mushrooms there is at least one camelina(or more) the number of mushrooms must be less than or equal to .

It follows that the number of saffron milk caps is greater than or equal to .

As among any 20 mushrooms at least one mushroom(or more), the number of saffron milk caps must be less than or equal to

Then we got that, on the one hand, the number of mushrooms is greater than or equal to 19 , and on the other hand, less than or equal to 19 .

Therefore, the number of mushrooms equals 19.

Answer: 19.

Problem number 6047.

Sasha invited Petya to visit, saying that he lives in the seventh entrance in apartment No. 333, but he forgot to say the floor. Approaching the house, Petya discovered that the house had nine floors. What floor does Sasha live on? (The number of apartments on each floor is the same, the numbers of apartments in the building start from one.)

Let on each floor of apartments.

Then the number of apartments in the first six entrances is

Find the maximum natural value that satisfies the inequality ( - the number of the last apartment in the sixth entrance, and it is less than 333.)

From here

The number of the last apartment in the sixth entrance -

The seventh entrance starts from the 325th apartment.

Therefore, apartment 333 is on the second floor.

Answer: 2

Problem number 6060.

On the surface of the globe, 17 parallels and 24 meridians were drawn with a felt-tip pen. Into how many parts do the lines drawn divide the surface of the globe? A meridian is an arc of a circle that connects the North and South Poles. Parallel is a circle lying in a plane parallel to the plane of the equator..

Imagine a watermelon that we cut into pieces.

Having made two cuts from the top point to the bottom (drawing two meridians), we will cut the watermelon into two slices. Therefore, after making 24 cuts (24 meridians), we will cut the watermelon into 24 slices.

Now we will cut each slice.

If we make 1 transverse cut (parallel), then we will cut one slice into 2 parts.

If we make 2 transverse cuts (parallels), then we will cut one slice into 3 parts.

So, having made 17 cuts, we will cut one slice into 18 parts.

So, we cut 24 slices into 18 pieces, and got a piece.

Therefore, 17 parallels and 24 meridians divide the surface of the globe into 432 parts.

Answer: 432.

Problem #6069

On the stick are marked transverse lines of red, yellow and green. If you saw the stick along the red lines, you get 5 pieces, if along the yellow lines - 7 pieces, and if along the green lines - 11 pieces. How many pieces will you get if you cut a stick along the lines of all three colors?

If you make 1 cut, you get 2 pieces.

If you make 2 cuts, you get 3 pieces.

In the general case: if you make cuts, you get a piece.

Back: to get pieces, you need to make a cut.

Find the total number of lines along which the stick was cut.

If you cut the stick along the red lines, you get 5 pieces - therefore, there were 4 red lines;

if on yellow - 7 pieces - therefore, there were 6 yellow lines;

and if on green - 11 pieces - therefore, there were 10 green lines.

Hence the total number of lines is . If you cut the stick along all the lines, you get 21 pieces.

Answer: 21.

Problem #9626.

There are four gas stations on the ring road: A, B, B, and D. The distance between A and B is 50 km, between A and C is 40 km, between C and D is 25 km, between D and A is 35 km (all distances are measured along the ring road in the shortest direction). Find the distance between B and C.

Let's see how gas stations can be located. Let's try to arrange them like this:


With such an arrangement, the distance between G and A cannot be equal to 35 km.

Let's try this:


With this arrangement, the distance between A and B cannot be 40 km.

Consider this option:


This option satisfies the condition of the problem.

Answer: 10.

Problem #10041.

The list of tasks of the quiz consisted of 25 questions. For each correct answer, the student received 7 points, for an incorrect answer, 9 points were deducted from him, and if there was no answer, they were given 0 points. How many correct answers were given by the student who scored 56 points, if it is known that he was wrong at least once?

Let the student give correct answers and incorrect ones ( ). Since there may have been more questions that he answered, we get the inequality:

In addition, according to the condition

Since a correct answer adds 7 points, and an incorrect answer subtracts 9, and the student ends up with 56 points, we get the equation:

This equation must be solved in integers.

Since 9 is not divisible by 7, it must be divisible by 7.

Let , then .

In this case, all conditions are met.

Problem #10056.

The rectangle is divided into four small rectangles by two straight cuts. The areas of three of them, starting from the top left and going clockwise, are 15, 18, 24. Find the area of ​​the fourth rectangle.


The area of ​​a rectangle is equal to the product of its sides.

The yellow and blue rectangles have a common side, so the ratio of the areas of these rectangles is equal to the ratio of the lengths of the other sides (not equal to each other).

The white and green rectangles also have a common side, so the ratio of their areas is equal to the ratio of other sides (not equal to each other), that is, the same ratio:

By the property of proportion, we get

From here.

Problem #10071.

The rectangle is divided into four small rectangles by two straight cuts. The perimeters of three of them, starting from the top left and going clockwise, are 17, 12, 13. Find the perimeter of the fourth rectangle.


The perimeter of a rectangle is equal to the sum of the lengths of all its sides.

Let us designate the sides of the rectangles as shown in the figure and express the perimeters of the rectangles in terms of the indicated variables. We get:

Now we need to find what the value of the expression is.

Subtract the second equation from the third equation and add the third. We get:

Simplify the right and left sides, we get:

So, .

Answer: 18.

Problem #10086.

The table has three columns and several rows. Each cell of the table was placed with a natural number so that the sum of all the numbers in the first column is 72, in the second - 81, in the third - 91, and the sum of the numbers in each row is greater than 13, but less than 16. How many rows are there in the table?

Let's find the sum of all the numbers in the table: .

Let the number of rows in the table be .

According to the condition of the problem, the sum of numbers in each line more than 13 but less than 16.

Since the sum of the numbers is a natural number, only two natural numbers satisfy this double inequality: 14 and 15.

If we assume that the sum of the numbers in each row is 14, then the sum of all the numbers in the table is , and this sum satisfies the inequality .

If we assume that the sum of the numbers in each row is 15, then the sum of all the numbers in the table is , and this number satisfies the inequality .

So, a natural number must satisfy the system of inequalities:

The only natural that satisfies this system is

Answer: 17.

It is known about the natural numbers A, B and C that each of them is greater than 4 but less than 8. They guessed a natural number, then multiplied it by A, then added it to the resulting product B and subtracted C. It turned out 165. What number was guessed?

Integers A, B and C can be equal to the numbers 5, 6 or 7.

Let the unknown natural number be .

We get: ;

Let's consider various options.

Let A=5. Then B=6 and C=7, or B=7 and C=6, or B=7 and C=7, or B=6 and C=6.

Let's check: ; (one)

165 is divisible by 5.

The difference between the numbers B and C is either equal to or equal to 0 if these numbers are equal. If the difference is , then equality (1) is impossible. Therefore, the difference is 0 and

Let A=6. Then B=5 and C=7, or B=7 and C=5, or B=7 and C=7, or B=5 and C=5.

Let's check: ; (2)

The difference between the numbers B and C is either equal to or equal to 0 if these numbers are equal. If the difference is equal to or 0, then equality (2) is impossible, since it is an even number, and the sum (165 + even number) cannot be an even number.

Let A=7. Then B=5 and C=6, or B=6 and C=5, or B=6 and C=6, or B=5 and C=5.

Let's check: ; (3)

The difference between the numbers B and C is either equal to or equal to 0 if these numbers are equal. The number 165, when divided by 7, gives a remainder of 4. Therefore, it is also not divisible by 7, and equality (3) is impossible.

Answer: 33

Several consecutive pages fell out of the book. The number of the last page before the dropped sheets is 352, the number of the first page after the dropped sheets is written in the same numbers, but in a different order. How many sheets fell out?

Obviously, the number of the first page after the dropped sheets is greater than 352, so it can be either 532 or 523.

Each dropped sheet contains 2 pages. Therefore, an even number of pages fell out. 352 is an even number. If we add an even number to an even number, we get an even number. Therefore, the number of the last dropped page is an even number, and the number of the first page after the dropped sheets must be odd, that is, 523. Therefore, the number of the last dropped page is 522. Then it fell sheets.

Answer: 85

Masha and the Bear ate 160 cookies and a jar of jam, starting and finishing at the same time. At first, Masha ate jam, and the Bear ate cookies, but at some point they changed. The bear eats both three times faster than Masha. How many cookies did the Bear eat if they ate the same amount of jam?

If Masha and the Bear ate jam equally, and the bear ate three times as much jam per unit time, then he ate jam three times less than Masha. In other words, Masha ate jam three times longer than the Bear. But while Masha was eating jam, the bear was eating cookies. Therefore, the bear ate cookies three times longer than Masha. But the Bear, moreover, ate three times more cookies per unit time than Masha, therefore, in the end, he ate 9 times more cookies than Masha.

Now it's easy to write an equation. Let Masha eat the cookies, then the Bear ate the cookies. Together they ate cookies. we get the equation:

Answer: 144

On the counter of the flower shop there are 3 vases with roses: orange, white and blue. To the left of the orange vase are 15 roses, to the right of the blue vase are 12 roses. There are 22 roses in total in vases. how many roses are in the orange vase?

Since 15+12=27, and 27>22, therefore, the number of flowers in one vase was counted twice. And it's a white vase, because it's supposed to be the vase that's to the right of the blue one and to the left of the orange one. So the vases are in this order:

From here we get the system:

Subtracting the first equation from the third equation, we get O = 7.

Answer: 7

Ten poles are interconnected by wires so that exactly 8 wires extend from each pole. how many wires are strung between these ten pillars?

Decision

Let's simulate the situation. Suppose we have two poles, and they are interconnected by wires so that exactly 1 wire leaves each pole. Then it turns out that 2 wires depart from the poles. But we have this situation:


That is, despite the fact that 2 wires depart from the poles, only one wire is stretched between the poles. This means that the number of extended wires is two times less than the number of outgoing ones.

We get: - the number of outgoing wires.

Number of wires stretched.

Answer: 40

Of the ten countries, seven have signed a friendship treaty with exactly three other countries, and each of the remaining three with exactly seven. How many contracts were signed in total?

This task is similar to the previous one: two countries sign one common treaty. Each contract has two signatures. That is, the number of signed agreements is half as much as the number of signatures.

Find the number of signatures:

Find the number of signed contracts:

Answer: 21

Three rays emanating from the same point divide the plane into three different angles, measured in integer degrees. The largest angle is 3 times the smallest. How many values ​​can the average angle take?

Let the smallest angle be , then the largest angle is . Since the sum of all angles is , the mean angle is .


The average angle must be greater than the smallest and less than the largest angle.

We get a system of inequalities:

Therefore, it takes values ​​in the range from 52 to 71 degrees, that is, all possible values.

Answer: 20

Misha, Kolya and Lesha are playing table tennis: the player who loses the game gives way to the player who did not participate in it. As a result, it turned out that Misha played 12 games, and Kolya - 25. How many games did Lesha play?

Decision

It should be explained how the tournament is organized: the tournament consists of a fixed number of games; the player who lost in this game gives way to a player who did not participate in this game. Following the results of the next game, the player who did not take part in it takes the place of the loser. Therefore, each player takes part in at least one of two consecutive games.

Let's find how many games there were.

Since Kolya played 25 games, therefore, at least 25 games were played in the tournament.

Misha played 12 games. Since he definitely took part in every second game, therefore, no more than games were played. That is, the tournament consisted of 25 games.

If Misha played 12 games, then Lesha played the remaining 13.

Answer: 13

At the end of the quarter, Petya wrote down in a row all his marks for one of the subjects, there were 5 of them, and put multiplication signs between some of them. The product of the resulting numbers turned out to be 3495 . What mark does Petya get in a quarter in this subject if the teacher puts only marks 2, 3, 4 or 5 and the final mark in the quarter is the arithmetic average of all current marks, rounded according to the rounding rules? (For example, 3.2 rounds up to 3; 4.5 rounds up to 5; 2.8 rounds up to 3)

Let's decompose 3495 into prime factors. The last digit of the number is 5, so the number is divisible by 5; The sum of the digits is divisible by 3, so the number is divisible by 3.

Got that

Therefore, Petya's estimates are 3, 5, 2, 3, 3. Let's find the arithmetic mean:

Answer: 3

The arithmetic mean of 6 different natural numbers is equal to 8. By how much should the largest of these numbers be increased so that their arithmetic mean becomes 1 more?

The arithmetic mean is equal to the sum of all numbers divided by their number. Let the sum of all numbers be . By the condition of the problem , therefore .

The arithmetic mean has increased by 1, that is, it has become equal to 9. If one of the numbers has been increased by , then the sum has increased by and has become equal to .

The number of numbers has not changed and is equal to 6.

We get the equality:

Consider such a task plan. We have the following conditions:

Total:N

From A pieces at least 1 of another type, and from B pieces at least 1 of the first type

Then: (A-1) is the minimum quantity of the first type, and (B-1) is the second.

After we do the check: (A-1) + (B-1) \u003dN.

EXAMPLE

AT

DECISION

So: we have 35 fish in total (perch and roach)

Consider the conditions: among any 21 fish there is at least one roach, then there is at least 1 roach in this condition, therefore (21-1) = 20 is the minimum of perches. Among any 16 fish - at least one perch, arguing similarly, (16-1) = 15 - this is the minimum of roaches. Now we check: 20 + 15 = 35, that is, we got the total number of fish, which means 20 perches and 15 roaches.

ANSWER: 15 roaches

    Quiz and number of correct answers

The list of tasks of the quiz consisted of A questions. For each correct answer, the student received a points, for an incorrect answer they were deducted from him.bpoints, and in the absence of a response, 0 points were given. How many correct answers were given by the student who scoredNpoints if it is known that at least once he was mistaken?

We know how many points he earned, we know the price of a correct and incorrect answer. Based on the fact that at least one wrong answer was given, then the number of points for correct answers should exceed the number of penalty points byNpoints. Let there be x correct answers and y incorrect answers, then:

a*x= N+ b* y

x=(N+ b* y)/a

from this equality it is clear that the number in brackets must be a multiple of a. With this in mind, we can evaluate y (it is also an integer). It should be noted that the number of correct and incorrect answers should not exceed the total number of questions.

EXAMPLE

DECISION:

we introduce the designations (for convenience) x - correct, y - incorrect, then

5*x=75+11*y

X=(75+11*y)/5

Since 75 is divisible by five, then 11*y must also be divisible by five. Therefore, y can take multiples of five (5, 10, 15, etc.). take the first value y=5 then x=(75+11*5)/5=26 total questions 26+5=31

Y=10 x=(75+11*10)=37 total answers 37+10= 47 (more than questions) does not fit.

So in total there were: 26 correct and 5 incorrect answers.

ANSWER: 26 correct answers

    What floor?

Sasha invited Petya to visit, saying that he lives in a stairwell in apartment No.N, and I forgot to say the floor. Approaching the house, Petya discovered that the housey-storey. What floor does Sasha live on? (On all floors, the number of apartments is the same, the numbers of apartments in the building start from one.)

DECISION

According to the condition of the problem, we know the apartment number, the entrance and the number of floors in the house. Based on these data, it is possible to make an estimate of the number of apartments per floor. Let x be the number of apartments per floor, then the following condition must be met:

A*y*x must be greater than or equalN

From this inequality we estimate x

To begin with, we take the minimum integer value x, let it be equal to c, and check: (a-1) * y * c is less thanN, and a*y*s is greater than or equal toN.

Having chosen the value x we ​​need, we can easily calculate the floor (c): in = (N-( a-1)* c)/ c, and in is an integer and getting a fractional value, we take the nearest integer (in a big way)

EXAMPLE

DECISION

Let's estimate the number of apartments on the floor: 7*7*x is greater than or equal to 462, hence x is greater than or equal to 462/(7*7)=9.42, which means the minimum x=10. We check: 6 * 7 * 10 = 420 and 7 * 7 * 10 = 490 as a result, we got that the apartment by number falls into this range. Now let's find the floor: (462-6*7*10)/10=4.2 means the boy lives on the fifth floor.

ANSWER: 5th floor

    Apartments, floors, entrances

All entrances of the house have the same number of floors, and all floors have the same number of apartments. At the same time, the number of floors in the house is greater than the number of apartments per floor, the number of apartments per floor is greater than the number of entrances, and the number of entrances is more than one. How many floors are there in a building if there are X apartments in total?

This type of task is based on the following condition: if in the house E - floors, P - entrances and K - apartments on the floor, then the total number of apartments in the house should be equal to E * P * K \u003d X. so we need to represent X as a product of three numbers not equal to 1 (according to the condition of the problem). To do this, we will decompose the number X into prime factors. Having made the decomposition and taking into account the conditions of the problem, we make a selection of the correspondence between the numbers and the conditions that are indicated in the problem.

EXAMPLE

DECISION

Let's represent the number 105 as a product of prime factors

105=5*7*3, now let's return to the condition of the problem: since the number of floors is the largest, it is 7, the number of apartments on the floor is 5, and the entrances are 3.

ANSWER: entrances - 7, apartments on the floor - 5, entrances - 3.

    Exchange

AT

For a gold coins, get from silver and copper;

For x silver coins get in gold and with 1 copper.

Nicholas had only silver coins. After the exchange office, he had fewer silver coins, no gold coins, but C copper coins appeared. By how much did Nicholas's number of silver coins decrease?

There are two exchange schemes in the exchange point:

EXAMPLE

AT The exchange office can perform one of two operations:

DECISION

5 gold=4 silver+1 copper

10 silver=7 gold+1 copper

since no gold coins appeared, we need an exchange scheme without gold coins. Therefore, the number of gold coins must be equal in both cases. We need to find the least common multiple of the numbers 5 and 7, and bring our gold in both cases to it:

35 gold=28 silver+7 copper

50 silver=35 gold+5 copper

as a result we get

50 silver=28 silver+12 copper

We found an exchange scheme bypassing gold coins, now we need, knowing the number of copper coins, to find how many times such an operation was performed

N=60/12=5

As a result, we get

250 silver=140 silver+60 copper

Substituting, and having received the final exchange, we will find how much silver was changed. So - the number decreased by 250-140=110

ANSWER for 110 coins

6. THE GLOBE

On the surface of the globe, the x parallels and the y meridian are drawn with a marker. Into how many parts did the drawn lines divide the surface of the globe? (a meridian is an arc of a circle connecting the North and South poles, and a parallel is the boundary of a section of a globe by a plane parallel to the plane of the equator).

DECISION:

Since the parallel is the boundary of the section of the globe by a plane, then one will break the globe into 2 parts, two into three parts, x into x + 1 parts

The meridian is an arc of a circle (more precisely, a semicircle) and the meridian breaks the surface into y parts, therefore the total will be (x + 1) * y parts.

EXAMPLE

After similar reasoning, we get:

(30+1)*24=744 (parts)

ANSWER: 744 parts

7. CUTTINGS

On the stick are marked transverse lines of red, yellow and green. If you cut a stick along the red lines, you get A pieces, if along the yellow lines - B pieces, and if along the green lines - From the pieces. How many pieces will you get if you cut a stick along the lines of all three colors?

DECISION

For the solution, we take into account that the number of pieces is 1 more than the number of cuts. Now you need to find how many lines are marked on the stick. We get red (A-1), yellow - (B-1), green - (C-1). Having found the number of lines of each color and summing them up, we get the total number of lines: (A-1) + (B-1) + (C-1). We add one to the resulting number (since the number of pieces is one more than the number of cuts), we get the number of pieces if we cut along all lines.

EXAMPLE

On the stick are marked transverse lines of red, yellow and green. If you saw the stick along the red lines, you get 7 pieces, if along the yellow lines - 13 pieces, and if along the green lines - 5 pieces. How many pieces will you get if you cut a stick along the lines of all three colors?

DECISION

Finding the number of lines

Reds: 7-1=6

Yellow: 13-1=12

Greens: 5-1=4

Total number of lines: 6+12+4=22

Then the number of pieces: 22+1=23

ANSWER: 23 pieces

8. COLUMNS AND ROWS

AT each cell of the table was placed according to a natural number so that the sum of all the numbers in the first column is equal to C1, in the second - C2, in the third - C3, and the sum of the numbers in each row is greater than Y1, but less than Y2. How many rows are in the table?

DECISION

Since the numbers in the cells of the table do not change, the sum of all the numbers in the table is: C=C1+C2+C3.

Now let's pay attention to the fact that the table consists of natural numbers, which means that the sum of numbers by rows must be integers and be in the range from (Y1 + 1) to (Y2-1) (since the sum of rows is strictly limited). Now we can estimate the number of rows:

C / (Y1 + 1) - the maximum number

C / (U2-1) - the minimum amount

EXAMPLE

AT The table has three columns and several rows. AT

DECISION

Find the sum of the table

С=85+77+71=233

Let's define the limits of the sum of rows

12+1=13 – minimum

15-1=14 - maximum

Estimate the number of rows in the table

233/13=17.92 maximum

233/14=16.64 minimum

Within these limits there is only one whole number - 17

ANSWER: 17

9. REFUELING AT THE RING

and D. The distance between A and B - 35 km, between A and B - 20 km, between B and G - 20 km, between G and A and V.

DECISION

Having carefully read the problem, we will notice that in practice the circle is divided into three arcs AB, VG and AG. Based on this, we will find the length of the entire circle (ring). For this task, it is equal to 20+20+30=70 (km).

Now, having placed all the points on the circle and signed the lengths of the corresponding arcs, it is easy to determine the required distance. In this problem BV=AB-AB, i.e. BV=35-20=15

ANSWER: 15 km

10. COMBINATIONS

DECISION

To solve this type of problem, remember what a factorial is.

Factorial of a numberN! called the product of consecutive numbers from 1 toN, that is, 4!=1*2*3*4.

Now back to the task. Find the total number of cubes: 3+1+1=5. Since we have three cubes of the same color, the total number of cubes can be found using the formula 5!/3! We get (5*4*3*2*1)/(1*2*3)=5*4=20

ANSWER: 20 ways to arrange

11 . WELLS

The owner agreed with the workers that they would dig a well for him under the following conditions: for the first meter he would pay them X rubles, and for each next meter - Y rubles more than for the previous one. How many rubles will the owner have to pay to the workers if they dig a well deepNmeters?

DECISION:

Since the owner increases the price for each meter, he will pay for the second (X + Y), for the third - (X + 2Y), for the fourth (X + 3Y), etc. It is not difficult to see that this payment system resembles an arithmetic progression, where a1 = X,d= Y, n= N. Then

Payment for work is nothing but the sum of this progression:

S= ( (2a+d(n-1))/2) n

EXAMPLE:

DECISION

Based on the above, we geta1=4200

d=1300

n=11

Substituting this data into our formula, we get

S=((2*4200+1300(11-1)/2)*11=((8400+13000)/2)*11=10700*11=117700

ANSWER: 117700

12 . POSTS AND WIRES

X poles connected by wires, so that exactly Y wires extend from each. How many wires are strung between the poles?

DECISION

Find how many gaps between the pillars. Between two there is one gap, between three - two, between four - 3, between X - (X-1).

At each gap Y wires, then (X-1) * Y is the total wires between the poles.

EXAMPLE

The ten poles are connected by wires, so that exactly 6 wires come out of each. How many wires are strung between the poles?

DECISION

Returning to the previous notation, we get:

X=9 Y=6

Then we get (9-1)*6=8*6=48

ANSWER: 48

13. SAWING BOARDS AND LOGS

There were several logs. They made X cuts and it turned out for chumps. How many logs were cut?

DECISION

When solving, let's make one remark: some problems do not always have a mathematical solution.

Now to the task. When deciding, it must be taken into account that there are more than one logs and when sawing each log, = 1 piece is obtained.

This type of problem is more convenient to solve by the selection method:

Let there be two logs, then the pieces will be 13 + 2 = 15

Take three we get 13+3=16

And here you can see the dependence that the number of cuts and pieces increases in the same way, that is, the number of logs that need to be cut is equal to Y-X

EXAMPLE

There were several logs. We made 13 cuts and got 20 chubachki. How many logs were cut?

DECISION

Returning to our reasoning, we can pick up, or you can just 20-13 \u003d 7 means only 7 logs

Answer 7

14 . LOSSED PAGES

Several pages fell out of the book. The first of the dropped pages has the number X, and the number of the last one is written in the same numbers in some other order. How many pages fell out of the book?

DECISION

The numbering of pages that have fallen out starts with an odd number and must end with an even number. Therefore, we, knowing that the number of the last dropped out is written in the same digits, that the first dropped out we know its last digit. By permuting the remaining digits and, given that the page numbering must be greater than the first one, we get its number. Knowing the page numbers, you can calculate how many of them fell out, while taking into account that page X also fell out. So from the resulting number we must subtract the number (X-1)

EXAMPLE

Several pages fell out of the book. The first of the dropped pages has the number 387, and the number of the last one is written in the same numbers in some other order. How many pages fell out of the book?

DECISION

Based on our reasoning, we get that the number of the last dropped page should end with the number 8. So we have only two options for numbers, these are 378 and 738. 378 does not suit us because it is less than the number of the first dropped page, which means the last dropped out is 738.

738-(387-1)=352

ANSWER: 352

The following should be added: sometimes they ask you to indicate the number of sheets, then the number of pages should be divided in half.

15. FINAL GRADE

At the end of the quarter, Little Johnny wrote down his current marks in singing in a row and put a multiplication sign between some of them. The products of the resulting numbers turned out to be equal to X. What mark does Vovochka have in the singing quarter?

DECISION

When solving this type of problem, it must be taken into account that its estimates should be 2,3,4 and 5. Therefore, we need to decompose the number X into factors 2,3,4 and 5. Moreover, the remainder of the expansion should also consist of these numbers.

EXAMPLE1

At the end of the quarter, Little Johnny wrote down his current marks in singing in a row and put a multiplication sign between some of them. The products of the resulting numbers turned out to be equal to 2007. What mark does Vovochka have in the singing quarter?

DECISION

Let's factorize the number 2007

We get 2007=3*3*223

So his grades are: 3 3 2 2 3 now find the arithmetic mean of his grades for this set is 2.6 hence his grade is three (greater than 2.5)

ANSWER 3

EXAMPLE 2

At the end of the quarter, Vovochka wrote down all his marks in a row for one of the subjects, there were 5 of them, and put multiplication signs between some of them. The product of the resulting numbers turned out to be 690. What mark does Vovochka get in the quarter in this subject, if the teacher puts only marks 2, 3, 4 and 5 and the final mark in the quarter is the arithmetic average of all current marks, rounded according to the rounding rules? (For example: 2.4 rounds up to two; 3.5 rounds up to 4; and 4.8 rounds up to 5.)

DECISION

We factorize 690 so that the remainder of the decomposition consists of the numbers 2 3 4 5

690=3*5*2*23

Hence his scores: 3 5 2 2 3

Let's find the arithmetic mean of these numbers: (3+5+2+2+3)/5=3

This will be his assessment.

ANSWER: 3

16 . MENU

The restaurant menu has X types of salads, Y types of first courses, A types of second courses, and B types of dessert. How many salad, first, second and dessert lunch options can diners at this restaurant choose?

DECISION

When solving, we will cut the menu a little: let there be only a salad and then the first options will become (X * Y). Now let's add the second dish, the number of options increases by A times and becomes (X*Y*A). Now let's add dessert. The number of options will increase by B times

Now we get the final answer:

N=X*U*A*B

EXAMPLE

DECISION
Based on the above, we get:

N=6*3*5*4=360

ANSWER: 360

17 . WE DIVIDE WITHOUT RESIDUE

In this section, we will consider the tasks on a specific example, for greater clarity.

Since we have a product of consecutive numbers and there are more than 7 of them, then at least one must be divisible by 7. So we have a product, one of the factors of which is divisible by 7, therefore, the whole product is also divisible by seven, which means the remainder of the division will be equal to zero, or for the second problem, the number of factors must be equal to the divisor.

18.TOURISTS

We will also consider this type of tasks with a specific example.

First, let's define what we need to find: route time = ascent + rest + descent

Rest we know, now we need to find the time of ascent and descent

Reading the problem, we see that in both cases (ascent and descent), the time depends as an arithmetic progression, but we still do not know what height the ascent was, although it is not difficult to find:

H=(95-50)15+1=4

We have found the height of the rise, now we will find the rise time as the sum of an arithmetic progression: Tlift = ((2*50+15*(4-1))*4)/2=290 minutes

Similarly, we find, given that now the progression difference is -10. We get Tdesc=((2*60-10(4-1))*4)/2= 180 minutes.

Knowing all the components, you can calculate the total time of the route:

T route = 290 + 180 + 10 = 480 minutes or converting to hours (divided by 60) we get 8 hours.

ANSWER: 8 hours

19. RECTANGLES

There are two types of problems for rectangles: for perimeters and for areas.

To solve such a plan of tasks, it is easy to prove that when splitting any rectangle with two straight cuts, we get four rectangles for which the following relations will always hold:

P1+P2=P3+P4

S1*S2=S3*S4,

where R perimeter , S - square

Based on these relations, we can easily solve the following problems

19.1.Perimeters

DECISION

Based on the above, we get

24+16=28+X

X=(24+16)-28=12

ANSWER: 12

19.2 AREAS

The rectangle is divided into four small rectangles by two straight cuts. The areas of three of them, starting from the top left and going clockwise, are 18, 12, and 20. Find the area of ​​the fourth rectangle.

DECISION

For the resulting rectangles, the following should be performed:

18*20=12*X

Then X=(18*20)/12=30

ANSWER: 30

20. THERE-HERE

A snail crawls up a tree by A m in a day, and slides down by B m in a night. The height of a tree is C m. In how many days will a snail crawl to the top of a tree for the first time?

DECISION

In one day, a snail can rise to a height of (A-B) meters. Since she can climb height A in one day, she must overcome height (C-A) before the last ascent. Based on this, we get that it will rise (C-A) \ (A-B) + 1 (we add one since it rises to height A in one day).

EXAMPLE

DECISION

Returning to our reasoning, we get

(10-4)/(4-3)+1=7

REPLY in 7 days

It should be noted that in this way it is possible to solve problems of filling something when something comes in and something flows out.

21. STRAIGHT JUMPS

The grasshopper jumps along the coordinate line in any direction for a unit segment per jump. How many different points on the coordinate line are there that the grasshopper can reach after making X jumps, starting from the origin?

DECISION

Suppose that the grasshopper makes all jumps in one direction, then it will hit the point with coordinate X. Now it jumps forward for (X-1) jumps and one back: it hits the point with coordinate (X-2). Considering all his jumps in this way, you can see that he will be at points with coordinates X, (X-2), (X-4), etc. This dependence is nothing more than an arithmetic progression with a differenced\u003d -2 and a1 \u003d X, andan=- X. Then the number of members of this progression is the number of points in which it can be. Let's find them

an=a1+d(n-1)

X=X+d(n-1)

2X=-2(n-1)

n=X+1

EXAMPLE

DECISION

Based on the above findings, we get

10+1=11

ANSWER 11 points

TASKS FOR INDEPENDENT SOLUTION:

1. Every second a bacterium divides into two new bacteria. It is known that the entire volume of one glass of bacteria is filled in 1 hour. In how many seconds will the glass be half filled with bacteria?

2. On the stick are marked transverse lines of red, yellow and green. If you cut a stick along the red lines, you get 15 pieces, if along the yellow lines - 5 pieces, and if along the green lines - 7 pieces. How many pieces will you get if you cut a stick along the lines of all three colors?

3. The grasshopper jumps along the coordinate line in any direction by a single segment per jump. The grasshopper starts jumping from the origin. How many different points on the coordinate line are there that the grasshopper can reach after making exactly 11 jumps?

4. There are 40 mushrooms in the basket: mushrooms and milk mushrooms. It is known that among any 17 mushrooms there is at least one camelina, and among any 25 mushrooms at least one mushroom. How many mushrooms are in the basket?

5. Sasha invited Petya to visit, saying that he lives in the seventh entrance in apartment No. 462, but he forgot to say the floor. Approaching the house, Petya discovered that the house had seven floors. What floor does Sasha live on? (On all floors, the number of apartments is the same, the numbers of apartments in the building start from one.)

6. Sasha invited Petya to visit, saying that he lives in the eighth entrance in apartment No. 468, but he forgot to say the floor. Approaching the house, Petya discovered that the house had twelve floors. What floor does Sasha live on? (On all floors, the number of apartments is the same, the numbers of apartments in the building start from one.)

7. Sasha invited Petya to visit, saying that he lives in the twelfth entrance in apartment No. 465, but he forgot to say the floor. Approaching the house, Petya discovered that the house had five floors. What floor does Sasha live on? (On all floors, the number of apartments is the same, the numbers of apartments in the building start from one.)

8. Sasha invited Petya to visit, saying that he lives in the tenth entrance in apartment No. 333, but he forgot to say the floor. Approaching the house, Petya discovered that the house had nine floors. What floor does Sasha live on? (On all floors, the number of apartments is the same, the numbers of apartments in the building start from one.)

9. The trainer advised Andrey to spend 15 minutes on the treadmill on the first day of classes, and on each next lesson to increase the time spent on the treadmill by 7 minutes. How many sessions will Andrey spend on the treadmill for a total of 2 hours and 25 minutes if he follows the advice of the trainer?

10. The doctor prescribed the patient to take the medicine according to the following scheme: on the first day he should take 3 drops, and on each next day - 3 drops more than on the previous one. Having taken 30 drops, he drinks 30 drops of the medicine for another 3 days, and then reduces the intake by 3 drops daily. How many vials of medicine should a patient buy for the entire course of treatment if each contains 20 ml of medicine (which is 250 drops)?

11. The doctor prescribed the patient to take the medicine according to the following scheme: on the first day he should take 20 drops, and on each next day - 3 drops more than on the previous one. After 15 days of taking the patient takes a break of 3 days and continues to take the medicine according to the reverse scheme: on the 19th day he takes the same number of drops as on the 15th day, and then reduces the dose by 3 drops daily until the dosage becomes less than 3 drops per day. How many vials of medicine should a patient buy for the entire course of treatment if each contains 200 drops?

12. The product of ten consecutive numbers is divided by 7. What can be the remainder?

13. In how many ways can two identical red dice, three identical green dice and one blue dice be lined up?

14. A full bucket of water with a volume of 8 liters is poured into a tank with a volume of 38 liters every hour, starting at 12 o'clock. But there is a small gap in the bottom of the tank, and 3 liters flow out of it in an hour. At what point in time (in hours) will the tank be completely filled.

15. What is the smallest number of consecutive numbers that must be taken so that their product is divisible by 7?

16. As a result of the flood, the pit was filled with water up to a level of 2 meters. The construction pump continuously pumps out water, lowering its level by 20 cm per hour. Groundwater, on the contrary, raises the water level in the pit by 5 cm per hour. How many hours of pump operation will the water level in the pit drop to 80 cm?

17. The restaurant menu has 6 types of salads, 3 types of first courses, 5 types of second courses and 4 types of dessert. How many salad, first, second and dessert lunch options can diners at this restaurant choose?

18. An oil company is drilling a well for oil production, which, according to geological exploration, lies at a depth of 3 km. During the working day, drillers go 300 meters deep, but during the night the well “silts up” again, that is, it is filled with soil by 30 meters. How many working days will oil workers drill a well to the depth of oil?

19. What is the smallest number of consecutive numbers that must be taken so that their product is divisible by 9?

20.

for 2 gold coins get 3 silver and one copper;

for 5 silver coins, get 3 gold and one copper.

21. On the surface of the globe, 12 parallels and 22 meridians were drawn with a felt-tip pen. Into how many parts did the drawn lines divide the surface of the globe?

A meridian is an arc of a circle connecting the North and South Poles. A parallel is a circle lying in a plane parallel to the plane of the equator.

22. There are 50 mushrooms in the basket: mushrooms and milk mushrooms. It is known that among any 28 mushrooms there is at least one camelina, and among any 24 mushrooms at least one mushroom. How many mushrooms are in the basket?

23. A group of tourists overcame a mountain pass. They covered the first kilometer of the ascent in 50 minutes, and each next kilometer passed 15 minutes longer than the previous one. The last kilometer before the summit was completed in 95 minutes. After a ten-minute rest at the top, the tourists began their descent, which was more gentle. The first kilometer after the summit was covered in an hour, and each next one is 10 minutes faster than the previous one. How many hours did the group spend on the entire route if the last kilometer of the descent was covered in 10 minutes.

24. There are four gas stations on the ring road: A, B, C and D. The distance between A and B is 35 km, between A and C is 20 km, between C and D is 20 km, between D and A is 30 km (all distances measured along the ring road in the shortest direction). Find the distance between B and C. Give your answer in kilometers.

25. There are four gas stations on the ring road: A, B, C and D. The distance between A and B is 50 km, between A and C is 40 km, between C and D is 25 km, between D and A is 35 km (all distances measured along the ring road in the shortest direction). Find the distance between B and C.

26. There are 25 students in the class. Several of them went to the cinema, 18 people went to the theater, and 12 people went to the cinema and the theater. It is known that three did not go to the cinema or to the theater. How many people in the class went to the movies?

27. According to Moore's empirical law, the average number of transistors on a chip doubles every year. It is known that in 2005 the average number of transistors on a chip was 520 million. Determine how many millions of transistors on a chip were on average in 2003.

28. There are 24 seats in the first row of the cinema hall, and in each next row there are 2 more than in the previous one. How many seats are in the eighth row?

29. On the stick are marked transverse lines of red, yellow and green. If you cut a stick along the red lines, you get 5 pieces, if along the yellow lines - 7 pieces, and if along the green lines - 11 pieces. How many pieces will you get if you cut a stick along the lines of all three colors?

30. In a home appliance store, sales of refrigerators are seasonal. In January, 10 refrigerators were sold, and in the next three months, 10 refrigerators were sold. Since May, sales have increased by 15 units compared to the previous month. Since September, sales began to decrease by 15 refrigerators every month compared to the previous month. How many refrigerators did the store sell in a year?

31. In the exchange office, you can perform one of two operations:

1) for 3 gold coins get 4 silver and one copper;

2) for 6 silver coins, get 4 gold and one copper.

Nikola had only silver coins. After visiting the exchange office, he had fewer silver coins, no gold coins, but 35 copper coins appeared. By how much did Nikola's number of silver coins decrease?

32. Sasha invited Petya to visit, saying that he lives in the seventh entrance in apartment No. 462, but he forgot to say the floor. Approaching the house, Petya discovered that the house had seven floors. What floor does Sasha live on? (The number of apartments on each floor is the same, the numbers of apartments in the building start from one.)

33. All entrances of the house have the same number of floors, and each floor has the same number of apartments. At the same time, the number of floors in the house is greater than the number of apartments per floor, the number of apartments per floor is greater than the number of entrances, and the number of entrances is more than one. How many floors are there in a building if there are 110 apartments in total?

34. The grasshopper jumps along the coordinate line in any direction for a unit segment per jump. How many different points on the coordinate line are there that the grasshopper can reach after making exactly 6 jumps, starting from the origin?

35. There are 40 mushrooms in the basket: mushrooms and milk mushrooms. It is known that among any 17 mushrooms there is at least one camelina, and among any 25 mushrooms at least one mushroom. How many mushrooms are in the basket?

36. There are 25 mushrooms in the basket: mushrooms and milk mushrooms. It is known that among any 11 mushrooms there is at least one camelina, and among any 16 mushrooms at least one mushroom. How many mushrooms are in the basket?

37. There are 30 mushrooms in the basket: mushrooms and milk mushrooms. It is known that among any 12 mushrooms there is at least one camelina, and among any 20 mushrooms at least one mushroom. How many mushrooms are in the basket?

38. On the globe, 17 parallels (including the equator) and 24 meridians were drawn with a felt-tip pen. Into how many parts do the lines drawn divide the surface of the globe?

39. A snail crawls 4 m up a tree in a day, and slides 3 m in a night. The height of a tree is 10 m. In how many days will a snail crawl to the top of a tree for the first time?

40. A snail crawls 4 m up a tree in a day, and slides 1 m in a night. The height of a tree is 13 m. How many days does it take for a snail to crawl to the top of a tree for the first time?

41. The owner agreed with the workers that they would dig a well for him on the following terms: for the first meter he would pay them 4,200 rubles, and for each next meter - 1,300 rubles more than for the previous one. How much money will the owner have to pay the workers if they dig a well 11 meters deep?

42. The owner agreed with the workers that they were digging a well on the following terms: for the first meter he would pay them 3,500 rubles, and for each next meter - 1,600 rubles more than for the previous one. How much money will the owner have to pay the workers if they dig a well 9 meters deep?

43. There are 45 mushrooms in the basket: mushrooms and milk mushrooms. It is known that among any 23 mushrooms there is at least one camelina, and among any 24 mushrooms at least one mushroom. How many mushrooms are in the basket?

44. There are 25 mushrooms in the basket: mushrooms and milk mushrooms. It is known that among any 11 mushrooms there is at least one camelina, and among any 16 mushrooms at least one mushroom. How many mushrooms are in the basket?

45. The list of tasks of the quiz consisted of 25 questions. For each correct answer, the student received 7 points, for an incorrect answer, 10 points were deducted from him, and if there was no answer, they were given 0 points. How many correct answers were given by the student who scored 42 points, if it is known that he was wrong at least once?

46. On the stick are marked transverse lines of red, yellow and green. If you saw the stick along the red lines, you get 5 pieces, if along the yellow lines - 7 pieces, and if along the green lines - 11 pieces. How many pieces will you get if you cut a stick along the lines of all three colors?

47. A snail crawls up a tree 2 m in a day, and slides down 1 m in a night. The height of the tree is 11 m. How many days will it take for a snail to crawl from the base to the top of the tree?

48. A snail crawls 4 m up a tree in a day, and slides 2 m in a night. The height of a tree is 14 m. How many days will it take for a snail to crawl from the base to the top of the tree?

49. The rectangle is divided into four smaller rectangles by two straight cuts. The perimeters of three of them, starting from the top left and proceeding clockwise, are 24, 28 and 16. Find the perimeter of the fourth rectangle.

50. In the exchange office, you can perform one of two operations:

1) for 2 gold coins get 3 silver and one copper;

2) for 5 silver coins, get 3 gold and one copper.

Nicholas had only silver coins. After several visits to the exchange office, he had fewer silver coins, no gold coins, but 50 copper coins appeared. By how much did Nicholas's number of silver coins decrease?

51. The rectangle is divided into four smaller rectangles by two straight cuts. The perimeters of three of them, starting from the top left and proceeding clockwise, are 24, 28 and 16. Find the perimeter of the fourth rectangle.

52. In the exchange office, you can perform one of two operations:

1) for 4 gold coins get 5 silver and one copper;

2) for 7 silver coins, get 5 gold and one copper.

Nicholas had only silver coins. After several visits to the exchange office, he had fewer silver coins, no gold coins, but 90 copper coins appeared. By how much did Nicholas's number of silver coins decrease?

53. All entrances of the house have the same number of floors, and each floor has the same number of apartments. At the same time, the number of entrances of the house is less than the number of apartments per floor, the number of apartments per floor is less than the number of floors, the number of entrances is more than one, and the number of floors is not more than 24. How many floors are there in a house if it has only 156 apartments?

54. AT There are 26 students in the class. Several of them listen to rock, 14 people listen to rap, and only three listen to both rock and rap. It is known that four do not listen to either rock or rap. How many people in the class listen to rock?

55. AT There are 35 fish in the cage: perches and roaches. It is known that among any 21 fish there is at least one roach, and among any 16 fish there is at least one perch. How many roaches are in the garden?

56. On the surface of the globe, 30 parallels and 24 meridians were drawn with a marker. Into how many parts did the drawn lines divide the surface of the globe? (a meridian is an arc of a circle connecting the North and South poles, and a parallel is the boundary of a section of a globe by a plane parallel to the plane of the equator).

57. AT prehistoric exchange office could do one of two things:
- For 2 skins of a cave lion, get 5 skins of a tiger and 1 skin of a wild boar;
- For 7 tiger skins, get 2 cave lion skins and 1 wild boar skin.
Un, the son of the Bull, had only the skins of a tiger. After several visits to the exchange office, the tiger skins did not increase, the cave lion skins did not appear, but 80 wild boar skins appeared. By how much did Un, the Bull's son, have the number of tiger skins reduced in the end?

58. AT military unit 32103 has 3 types of salad, 2 types of first course, 3 types of second course and a choice of compote or tea. How many lunch options, which must consist of one salad, one first course, one second course and one drink, can the soldiers of this military unit choose?

59. A snail crawls 5 meters up a tree during the day, and slides down 3 meters during the night. The height of the tree is 17 meters. On what day will the snail crawl to the top of the tree for the first time?

60. In how many ways can three identical yellow cubes, one blue cube and one green cube be placed in a row?

61. The product of sixteen consecutive natural numbers is divided by 11. What can be the remainder of the division?

62. Every minute a bacterium divides into two new bacteria. It is known that bacteria fill the entire volume of a three-liter jar in 4 hours. How many seconds does it take bacteria to fill a quarter of a jar?

63. The list of tasks of the quiz consisted of 36 questions. For each correct answer, the student received 5 points, for an incorrect answer, 11 points were deducted from him, and if there was no answer, they were given 0 points. How many correct answers were given by the student who scored 75 points, if it is known that he was wrong at least once?

64. The grasshopper jumps along a straight road, the length of one jump is 1 cm. First, he jumps 11 jumps forward, then 3 back, then again 11 jumps and then 3 jumps back, and so on, how many jumps he will make by the time when he first finds himself at a distance of 100 cm from the start.

65. On the stick are marked transverse lines of red, yellow and green. If you cut a stick along the red lines, you get 7 pieces, if along the yellow lines - 13 pieces, and if along the green lines - 5 pieces. How many pieces will you get if you cut a stick along the lines of all three colors?

66. AT The exchange office can perform one of two operations:
for 2 gold coins get 3 silver and one copper;
for 5 silver coins, get 3 gold and one copper.
Nicholas had only silver coins. After several visits to the exchange office, he had fewer silver coins, no gold coins, but 50 copper coins appeared. By how much did Nicholas's number of silver coins decrease?

67. The rectangle is divided into four smaller rectangles by two straight cuts.
The perimeters of three of them, starting from the top left and proceeding clockwise, are 24, 28 and 16. Find the perimeter of the fourth rectangle.

68. AT The exchange office can perform one of two operations:
1) for 4 gold coins get 5 silver and one copper;
2) for 7 silver coins, get 5 gold and one copper.
Nikola had only silver coins. After visiting the exchange office, he had fewer silver coins, no gold coins, but 90 copper coins appeared. By how much did the number of silver coins decrease?

69. A snail crawls 4 m up a tree in a day, and slides 2 m in a night. The height of a tree is 12 m. How many days will it take for a snail to crawl from the base to the top of the tree?

70. The list of tasks of the quiz consisted of 32 questions. For each correct answer, the student receives 5 points. 9 points were written off for the wrong answer, 0 points were given if there was no answer.
How many correct answers did the student who scored 75 points give if he was wrong at least 2 times?

71. The list of tasks of the quiz consisted of 25 questions. For each correct answer, the student received 7 points, for an incorrect answer, 10 points were deducted from him, and if there was no answer, they were given 0 points. How many correct answers were given by the student who scored 42 points, if it is known that he was wrong at least once?

72. The owner agreed with the workers that they would dig a well for him on the following terms: for the first meter he would pay them 4,200 rubles, and for each next meter - 1,300 rubles more than for the previous one. How many rubles will the owner have to pay to the workers if they dig a well 11 meters deep?

73. The rectangle is divided into four small rectangles by two straight cuts. The areas of three of them, starting from the top left and going clockwise, are 18, 12, and 20. Find the area of ​​the fourth rectangle.

74. The rectangle is divided into four small rectangles by two straight cuts. The areas of three of them, starting from the top left and going clockwise, are 12, 18, and 30. Find the area of ​​the fourth rectangle.

75. AT The table has three columns and several rows. AT each cell of the table was placed according to a natural number so that the sum of all the numbers in the first column is 85, in the second - 77, in the third - 71, and the sum of the numbers in each row is greater than 12, but less than 15. How many rows are there in the table?

76. The grasshopper jumps along the coordinate line in any direction for a unit segment per jump. How many different points on the coordinate line are there that the grasshopper can reach after making 10 jumps, starting from the origin?

77. Sasha invited Petya to visit, saying that he lives in the seventh entrance in apartment No. 462, but he forgot to say the floor. Approaching the house, Petya discovered that the house had seven floors. What floor does Sasha live on? (On all floors, the number of apartments is the same, the numbers of apartments in the building start from one.)

78. AT The exchange office can perform one of two operations:
for 2 gold coins get 3 silver and one copper;
for 7 silver coins get 3 gold and one copper.
Nicholas had only silver coins. After the exchange office, he did not have gold coins, but 20 copper coins appeared. By how much did Nicholas's number of silver coins decrease?

79. The grasshopper jumps along the coordinate line in any direction for a unit segment per jump. How many different points on the coordinate line are there that the grasshopper can reach after making 11 jumps, starting from the origin?

80. There are four gas stations on the ring road: A, B, C and D. The distance between A and B - 35 km, between A and B - 20 km, between B and G - 20 km, between G and A - 30 km (all distances are measured along the ring road in the shortest arc). Find the distance (in kilometers) between B and V.

81. AT The exchange office can perform one of two operations:
for 4 gold coins get 5 silver and one copper;
for 7 silver coins get 5 gold and one copper.
Nicholas had only silver coins. After the exchange office, he had fewer silver coins, there were no gold coins, but 90 copper coins appeared. By how much did Nicholas's number of silver coins decrease?

82. A grasshopper jumps along a coordinate line in any direction for a unit segment per jump. How many points on the coordinate line are there that the grasshopper can reach after making exactly 8 jumps, starting from the origin?

83. AT The exchange office can perform one of two operations:
for 5 gold coins get 4 silver and one copper;
For 10 silver coins, get 7 gold and one copper.
Nicholas had only silver coins. After the exchange office, he had fewer silver coins, no gold coins, but 60 copper coins appeared. By how much did Nicholas's number of silver coins decrease?

84. AT The exchange office can perform one of two operations:
for 5 gold coins get 6 silver and one copper;
for 8 silver coins get 6 gold and one copper.
Nicholas had only silver coins. After the exchange office, he had fewer silver coins, no gold coins, but 55 copper coins appeared. By how much did Nicholas's number of silver coins decrease?

85. All entrances of the house have the same number of floors, and all floors have the same number of apartments. At the same time, the number of floors in the house is greater than the number of apartments per floor, the number of apartments per floor is greater than the number of entrances, and the number of entrances is more than one. How many floors are there in a building if there are 105 apartments in total?

86. AT The exchange office can perform one of two operations:
1) for 3 gold coins get 4 silver and one copper;
2) for 7 silver coins, get 4 gold and one copper.
Nikola had only silver coins. After visiting the exchange office, he had fewer silver coins, no gold coins, but 42 copper coins appeared. By how much did Nikola's number of silver coins decrease?

ANSWERS